¿Un anillo giratorio de carga irradia energía o no?

La fórmula de Larmor sugiere que la potencia radiada por una carga acelerada no es cero. Pero sabemos que un anillo no conductor con carga uniforme que gira alrededor de su eje central no irradia. ¿Por qué esto es tan?

Esto es de alguna manera muy extraño para mí. Por ejemplo, supongamos que cargo una porción muy pequeña del anillo no conductor con una carga q. Ahora, cuando giro este anillo con una velocidad angular constante, debería haber radiación. Ahora, todo lo que tengo que hacer es rociar un poco de carga uniformemente en el anillo giratorio, ¡y la radiación simplemente cesa! ¿No es esto extraño? ¿Podemos negar la radiación simplemente rociando la carga con cuidado?

Explique por qué no hay radiación en este caso.

Algunas diferencias de opinión en otro foro. researchgate.net/post/…
Creo que se trata de interferencia. Quiero decir, si tienes cargas +Q y -Q una encima de la otra, no esperas que irradien. Sin embargo, debido a la linealidad de las ecuaciones de Maxwell, se pueden resolver las ecuaciones de Maxwell correspondientes usando aceleraciones +Q y -Q como término fuente por separado. Ambos irradian, pero con una diferencia de fase de 180. Entonces uno puede sumar estos, y la radiación se desvanece. Sin embargo, en su caso, el problema es más sutil. Pero en principio, uno debería hacer S = ( mi 1 + mi 2 ) × ( B 1 + B 2 ) en lugar de S = S 1 + S 2 = mi 1 × B 1 + mi 2 × B 2 .
@Farcher Leí las primeras respuestas en ese foro, y creo que todas están equivocadas o no entienden el punto. Por ejemplo, no se requiere un dipolo oscilante para la radiación.
No estoy familiarizado con el hecho de que un anillo giratorio cargado no irradia. ¿Puede proporcionar una referencia a eso?
@garyp, vea la primera respuesta aquí: physics.stackexchange.com/questions/13416/…
Parafraseando la pregunta: una partícula cargada que se mueve en un círculo irradiará, pero una distribución de carga continua no lo hará. ¿Cuál es la característica esencial responsable de la diferencia? ¿Cómo encaja la radiación de sincrotrón en la imagen?
La pregunta mejoraría si fuera explícito que el eje de rotación era el eje de simetría (lo que parece ser asumido por los comentarios y las respuestas, pero no parece indicarse en ninguna parte).
@dmckee, mencioné el 'eje central' en el primer párrafo. A pesar de que debería haber sido más explícito, no voy a cambiar nada ya que todos entendieron lo que quise decir.

Respuestas (3)

Para irradiar, debes tener una fluctuación con el tiempo.

Si toma una pequeña porción del anillo, la carga y luego la hace girar, la distribución de las cargas cambia con el tiempo.

Si el anillo está cargado uniformemente, sin corrientes adicionales, y haces que el anillo gire, entonces en esta situación (ideal) la distribución de cargas entre dos tiempos es siempre la misma. No habría manera de diferenciar la situación en t 1 y t 2 , por lo que no hay radiación. Supongo que si calcula los campos EM radiados por cada carga y los suma sobre el anillo, se cancelan.

Estoy hablando de una situación esencialmente clásica aquí
He verificado con múltiples fuentes, todas confirmando que no se irradia energía en este caso. Simplemente no entiendo por qué. He leído en griffiths sobre el vector puntiagudo y cómo en el límite r tiende a infinito, el flujo de S a través de una esfera de radio r, se toma como potencia radiada. Pero no pude probarlo en este caso, ya que las formas vectoriales no se manipulan tan fácilmente para obtener el vector de puntos.
consulte la página: 2, última línea, continuación. en la página 3.
@Lelouch Creo que entendí mal tu pregunta original entonces. Pero como explica el artículo, para irradiar, debe haber una fluctuación con el tiempo, en su situación, la distribución de cargas entre dos tiempos es siempre la misma, por lo que no hay radiación. Supongo que si calcula los campos EM radiados por cada carga y los suma sobre el anillo, se cancelan.
Creo que eso es cierto. También leí sobre otras cosas. Gracias de todas formas.

Los fotones son su propia antipartícula y pueden cancelarse. O dicho de otra manera, la luz es una onda y las ondas pueden interferir destructivamente. Con el anillo giratorio de carga, hay muchos campos radiantes, pero la simetría del problema les permite cancelarse destructivamente, por lo que no hay onda neta ni pérdida de potencia por la radiación. La radiación aquí es solo una onda EM que transporta energía.

Piense en una situación algo similar de la electrostática: si agrega carga eléctrica a un conductor perfecto, las cargas se organizan en la superficie y el campo eléctrico es 0 dentro del conductor. Hay muchos cargos con muchos campos, pero todos se cancelan. Esto es más fácil de calcular para una esfera, en caso de que quieras probarlo.

Déjame preguntar algo un poco sin relación. El hecho de que los electrones se organicen en un conductor para que sea equipotencial también se debe a que, en términos de energía de campo total, esta es la configuración de energía mínima (creo que es el teorema de Thomson). Si también queremos hacer una configuración de energía mínima de este anillo giratorio y darle la oportunidad de irradiar su energía y adquirir un estado de energía de campo más bajo, ¿por qué no lo hace?
@Lelouch: esa es una excelente pregunta. Creo que para comenzar a irradiar, las cargas primero tendrían que pasar a un estado asimétrico de mayor energía. Pero, ¿y si comienzan en el estado asimétrico? La potencia perdida por la radiación del ciclotrón no sería muy grande, y puedo imaginarme fácilmente que la energía podría perderse más rápido reorganizándola para que sea simétrica.

Abordé esta pregunta en mi respuesta ¿ Por qué las corrientes de bucle no producen luz? . A medida que divide la única partícula cargada en más y más partes mientras mantiene fijas la carga total y la corriente promedio, la distribución de la carga varía cada vez menos con el tiempo y la potencia radiada disminuye gradualmente hasta cero.